Rotationsstabilität eines rechteckigen Prismas

Ich habe etwas Merkwürdiges an der Drehung eines rechteckigen Prismas bemerkt. Wenn ich eine Kiste mit Höhe nehme Breite Tiefe und drehen Sie es um verschiedene Rotationsachsen in die Luft, einige Bewegungen scheinen stabiler als andere. Die 3 Achsen, die am besten veranschaulichen, was ich meine, sind:

(1) Durch den Massenmittelpunkt, parallel zur längsten Kastenkante.

(2) Durch den Schwerpunkt, parallel zur kürzesten Kastenkante.

(3) Durch den Schwerpunkt, parallel zum verbleibenden Kastenrand.

Es ist "einfach", die Box dazu zu bringen, sich sauber um (1) und (2) zu drehen, aber das Umdrehen der Box (3) führt normalerweise zu einer zusätzlichen Verdrehung neben der Drehung um (3), die ich zu erreichen versuche (offensichtlich a Ein "perfekter" Flip meinerseits würde dieses Verdrehen vermeiden, weshalb ich es eine Instabilität nenne). Wenn Sie nicht ganz sicher sind, wovon ich spreche, schnappen Sie sich eine Schachtel oder ein Buch mit 3 verschiedenen Seitenlängen und probieren Sie es aus (aber achten Sie darauf, dass nichts kaputt geht!).

Was ist das Besondere an Achse (3)?

Geben Sie hier die Bildbeschreibung einBild von Marsden und Ratiu .

Sie haben experimentell eine Tatsache entdeckt, die mathematisch in einem Oberstufen- oder Graduierten-Mechanikkurs gezeigt werden kann (Goldstein tut es; ich kann mich nicht erinnern, ob Marion und Thorten es getan haben). Ich werde keine Antwort schreiben, weil ich nie verstanden habe, „was das Besondere ist ich 2 ?" zu meiner eigenen Zufriedenheit, aber es ist ein bekanntes Verhalten.
Hm, hoffentlich hat jemand ein intuitives Verständnis entwickelt und kann mitteilen. Ich kann der Mathematik folgen, die das Phänomen erklärt, aber wie Sie habe ich immer noch keine Ahnung, warum ich 2 soll besonders sein.
Ich bin mir ziemlich sicher, dass es nichts mit dem Luftwiderstand zu tun hat, obwohl ich das Experiment nicht im Vakuum versucht habe, um es zu überprüfen ...
Cool, dass du das empirisch herausgefunden hast! Ich habe hier eine ziemlich einfache Erklärung geschrieben: lightandmatter.com/area1sn.html (Abschnitt 4.3.3).
Das ist eine ziemlich nette Analyse, möchten Sie sie in eine Antwort schreiben? Wenn nicht, könnte ich...
Für den Dzhanibekov-Effekt, das Tennisschläger-Theorem und das Intermediate-Axis-Theorem siehe z . B. physical.stackexchange.com/q/17504/2451 und darin enthaltene Links.
@BenCrowell Auf welcher Seite ist deine Erklärung?
Ein Video, das dies anhand eines Holzklotzes demonstriert: youtu.be/ms_lUMIwzFg

Antworten (4)

Das rechteckige Prisma ist ein starrer Körper. Die Bewegungsgleichungen eines starren Körpers um seinen Massenmittelpunkt sind gegeben durch: (Siehe zum Beispiel: Marsden und Ratiu , (Seite 6).

ich 1 Ω ˙ 1 = ( ich 2 ich 3 ) Ω 2 Ω 3
ich 2 Ω ˙ 2 = ( ich 3 ich 1 ) Ω 3 Ω 1
ich 3 Ω ˙ 3 = ( ich 1 ich 2 ) Ω 1 Ω 2

Wo Ω 1 , 2 , 3 sind die Winkelgeschwindigkeitskomponenten um die Körperachsen und ich 1 , 2 , 3 sind die entsprechenden Trägheitsmomente.

Da die Trägheitsmomente unterschiedlich sind, können wir ohne Beschränkung der Allgemeinheit annehmen, dass: ich 1 > ich 2 > ich 3 .

Tatsache ist, dass die stetige Bewegung um die Zwischenachse 2 ist nicht stabil, während die Bewegung um die beiden anderen Achsen stabil ist. Diese Tatsache wird von Marsden und Ratiu auf Seite 30 erklärt. Außerdem werden verschiedene andere Erklärungen in den Antworten auf eine verwandte Frage gegeben , die auf mathoverflow gestellt wurde. Hier beschreibe ich die Details einer linearisierten Stabilitätsanalyse.

Ein stationärer Zustand, in dem der Winkelgeschwindigkeitsvektor nur eine nicht verschwindende konstante Komponente hat, ist eine Lösung der Bewegungsgleichungen.

Zum Beispiel:

Ω 1 = Ω = c Ö n s t .
Ω 2 = 0
Ω 3 = 0

ist eine Lösung, die die Drehung um die erste Achse beschreibt. Ebenfalls

Ω 1 = 0
Ω 2 = Ω = c Ö n s t .
Ω 3 = 0

ist auch eine Lösung, die die Drehung um die zweite Achse beschreibt.

Jetzt können wir die Stabilität kleiner Störungen um diese Lösungen herum analysieren. Eine Störung der ersten Lösung ist gegeben durch:

Ω 1 = Ω + ϵ ω 1
Ω 2 = ϵ ω 2
Ω 3 = ϵ ω 3

Mit ϵ << 1 . Einsetzen in die Bewegungsgleichungen und Beibehalten nur der Terme bis zur ersten Potenz von ϵ , wir erhalten:

ich 2 ω ˙ 2 = ϵ Ω ( ich 3 ich 1 ) ω 3
ich 3 ω ˙ 3 = ϵ Ω ( ich 1 ich 2 ) ω 2

Bildet man die erste Ableitung der zweiten Gleichung nach der Zeit und setzt man die zweite Gleichung ein, erhält man:

ich 2 ich 3 ω ¨ 3 = ϵ 2 Ω 2 ( ich 3 ich 1 ) ( ich 1 ich 2 ) ω 3

Seit ich 3 < ich 1 und ich 1 > ich 2 , der Koeffizient auf der rechten Seite ist negativ und die Störung erfüllt eine harmonische Oszillatorbewegungsgleichung der Form:

ω ¨ 3 + k 2 ω 3 = 0

Durch Wiederholung der Störungsanalyse für die zweite Lösung (Rotation um die zweite Achse) erhalten wir:

ich 2 ich 3 ω ¨ 3 = ϵ 2 Ω 2 ( ich 2 ich 3 ) ( ich 1 ich 2 ) ω 3

Seit ich 3 < ich 2 und ich 1 > ich 2 , ist dieser Koeffizient nun negativ und die Lösung beschreibt einen harmonischen Oszillator mit negativer Federkonstante der Form:

ω ¨ 3 k 2 ω 3 = 0

Das ist eine instabile Störung.

Nun, das ist eine sehr klare Demonstration der stabilen/instabilen Bewegungen (+1) ... aber irgendeine Intuition darüber, was sie auslöst?
Dies ist die übliche Analyse und sie funktioniert einwandfrei. Aber es hinterlässt nie das gemeine Gefühl, dass ich es „verstehe“. Für mich ist es eine mathematische Tatsache ohne Intuition. Ich hoffe immer noch auf eine Antwort, die mir dieses "Ah ha!" Moment.
Der Link, den Ben Crowell in den Kommentaren zu der Frage gibt, macht einen besseren Job (er befindet sich in diesem Abschnitt lightandmatter.com/simplef.pdf ), als nur zu zeigen, dass er instabil ist, aber er gibt mir immer noch das Gefühl, dass ich ihn "verstehe". .
@dmckee: Vielleicht wissen Sie Terry Taos Antwort auf diese Frage zu Math Overflow zu schätzen: mathoverflow.net/questions/81960/… (auch am Ende von JoeHobbits Antwort verlinkt). Ich finde es ziemlich befriedigend (wenn auch ein bisschen peinlich, dass ein Mathematiker eine so klare, physikalische Antwort gibt). Es sind viele Worte, aber es lohnt sich, sie durchzugehen!

Geben Sie hier die Bildbeschreibung ein

Die der Mittellängenachse innewohnende Instabilität bzw 2 wie oben gezeigt, wird ausführlich in Marsden und Ratiu diskutiert , woher das Bild stammt.

Die instabile homokline Umlaufbahn, die die beiden instabilen Punkte verbindet, weist interessante Merkmale auf. Sie sind nicht nur interessant wegen der chaotischen Lösungen, die mit der Poincare-Melnikov-Methode in verschiedenen gestörten Systemen erhalten werden können (Ref), sondern bereits die Umlaufbahn selbst ist interessant, da ein starrer Körper, der um seine Mittelachse geworfen wird, eine interessante ( und unerwartet) halbe Verdrehung beim Erreichen des gegenüberliegenden Sattelpunktes, obwohl die Rotationsachse wieder da ist, wo sie war.

Die interessante Halbdrehung zeigt sich am besten im „ Dzhanibekov-Effekt “ und kann auch im „ Tennisschläger-Theorem “ gesehen werden.

Für diejenigen, die nicht verstehen, warum der Sattel entlang der Mittellängenachse zeigt 2 im obigen Bild ist instabil Betrachten Sie das folgende Bild:

Geben Sie hier die Bildbeschreibung ein Bildquelle

Die drei Achsen, die Sie beschreiben, sind jeweils vergleichbar mit:

  1. ~ eine Stange / Achse
  2. ~ ein Schwungrad
  3. ~ ein Propeller

Was ist Stabilität und warum sind zwei Achsen stabil, während die dritte instabil ist?

Stabilität bezieht sich auf eine "stabile" Schwingung, die harmonisch sein muss wie eine Masse auf einer Feder. Es gibt eine Rückstellkraft proportional zur Verschiebung.

F = k x = m a = m d 2 x d t 2                                         (1)
Für Winkelsituationen wird die Situation viel komplizierter, da das Drehmoment normal zur Rotationsebene ist.
τ = κ θ = ich ω ˙ = ich d 2 θ d t 2                                         (2)

Wenn es um eine stabile Achse eine außeraxiale Kraft gibt, gibt es zwei Drehmomentkomponenten: eine entlang der Hauptachse, die immer eine lineare Drehung verursacht, und eine zweite senkrecht zur Achse (etwa 1 oder beide der anderen Achsen), die in der Das Fehlen der Primärrotation (oder bei einer gleichmäßigen Massenverteilung) würde auch eine lineare Rotation verursachen. Es gibt also immer zwei Drehmomente; der erste groß, der zweite klein. Bei stabilen harmonischen Schwingungen wie bei stabilen Rotationsachsen gibt es eine dem Weg proportionale Rückstellkraft. Richard Feynman hat eine faszinierende Arbeit geleistet, um eine wackelnde Platte zu beschreiben , die doppelt so oft wackelt, wie sie sich dreht.

Lassen x ^ 1 , x ^ 2 ,und x ^ 3 Achsen entlang der längsten, mittleren bzw. kürzesten Achse des rechteckigen Prismas sein. Während einer stabilen Rotation (die auftritt, wenn die Primärrotation ca x ^ 1 , und x ^ 3 ) zeichnen die Sekundärachse(n) Kreise nach, wie von Feynman beschrieben .

Die Durchführung einer Analyse eines rechteckigen Prismas nach der von Feynman beschriebenen Methode wird sicherlich diese Rotation zeigen x ^ 2 erstellt eine Spirale anstelle eines Kreises.

Die Spirale auftreten

Stellen Sie sich vor, eine Scheibe, die sich um ihre eigene Achse dreht, ist sehr stabil: Die Differenz zwischen den Trägheitsmomenten um die anderen beiden Achsen ist Null: Sie ist sehr stabil. Ersetzen Sie nun die Scheibe "O" durch eine "X"-förmige Struktur, die sich entlang einer Achse senkrecht zur Ebene des X dreht. Die Rotation ist aus dem gleichen Grund wieder stabil. Schneiden Sie zwei Arme des X auf gegenüberliegenden Seiten ab und der gerade Stab dreht sich in stabilen Schwingungen weiter. Fügen Sie nun einen Draht entlang der Drehachse hinzu, der jedoch nur aus einer Seite der Stange herausragt. Plötzlich haben Sie den Dzhanibekov-Effekt, die instabil ist, genau wie das Verlängern des Stabs entlang der Rotationsachse, um eine Form zu bilden, die mit einem rechteckigen Prisma vergleichbar ist. Im Fall des Kabels ist es immer noch verwirrend, aber ich denke, es gibt einen Einblick in die Natur des Problems. Vor allem, wenn man bedenkt, dass ein Kreisel (sich drehende Scheibe mit asymmetrisch verlaufendem Draht) sehr stabil ist, ebenso wie ein X-förmiger Kreisel ... während ein Propellerkreisel nicht einmal wirklich ein Kreisel ist. Nehmen Sie also einen O-förmigen Kreisel, der sich in der Schwerelosigkeit dreht, und lassen Sie zufällig fast halbkreisförmige Stücke der Scheibe davonfliegen, sodass er sich in einen Propeller verwandelt. Jetzt wird das Trägheitsmoment um die Achse des rotierenden Propeller-"Blatts" (die längste Längsachse) stark reduziert, während gleichzeitig die Kreiselkraft drastisch reduziert wird.Es ist sinnvoll, dass diese (längste) Achse bis zu einem gewissen Grad zu einer "Achse der freien Rotation" wird ... wobei die Kreisel- oder Zentrifugalkräfte der sich drehenden "Klinge" beim Zurückklappen zur Welle hinzugefügt und dann von ihr abgezogen werden her im Dzhanibekov-Effekt . Der Unterschied zwischen der Länge der Achse mittlerer Länge 2 und die kürzeste Achse 1 hat die gleiche Funktion wie die Welle des propellerähnlichen Objekts im Dzhanibekov-Effekt: Insbesondere gibt und nimmt sie zentripetale Energie von der primären Rotationsachse ab 2 wie durch den Sattelpunkt dargestellt.

Beachten Sie auch, wie ein Kreisel, wenn er langsamer wird, in immer größeren Kreisen zu präzedieren beginnt, bis er umfällt. Ist das nur eine gyroskopische Präzession? Oder ist es das erste Anzeichen einer instabilen Schwingung, vergleichbar mit der Spiralspur der Achse beim Dzhanibekov-Effekt? Ich würde spekulieren, dass es ein bisschen von beidem ist: Die Oberseite ist wahrscheinlich keine perfekte Scheibe, und sobald das Wackeln beginnt, trägt die gyroskopische Präzession wahrscheinlich dazu bei.

Ich möchte hinzufügen, dass ein Y-förmiges Oberteil (60 Grad Abstand) einige besonders faszinierende Eigenschaften hat, da es Ähnlichkeiten sowohl mit einem Kasten als auch mit einem Propeller aufweist, aber ein Oberteil bleibt, weil die radiale Symmetrie es Kreiselkräften ermöglicht, die Achse mittlerer Länge zu stabilisieren. Wie von Ben Crowell in den Kommentaren hervorgehoben, wird dieser Effekt hier in Abschnitt 4.3.3, der den direkten Link zum PDF enthält, hier in schönen intuitiven Details erklärt . Ich habe die Erklärung dort wie folgt kopiert:

Für ein typisches, asymmetrisches Objekt müssen der Drehimpulsvektor und der Winkelgeschwindigkeitsvektor nicht parallel sein. Das heißt, nur für einen Körper, der Symmetrie um die Rotationsachse besitzt, gilt dies L = ich ω (das Rotationsäquivalent von p = m v ) für irgendeinen Skalar I.... (schicke Ableitung von:)

K = 1 2 L ω
.... Analysieren wir das Problem des sich drehenden Schuhs, das ich zu Beginn des Kapitels gestellt habe. Die dort genannten drei Rotationsachsen (vergleichbar mit einem rechteckigen Prisma) sind ungefähr die Hauptachsen des Schuhs. Während der Schuh in der Luft ist, wirken keine äußeren Drehmomente auf ihn, daher muss sein Drehimpulsvektor konstant sein. Das liegt jedoch im Bezugsrahmen des Raums. Der Hauptachsenrahmen ist am Schuh befestigt und stürzt wie verrückt mit. Im Rahmen der Hauptachse bleiben die kinetische Energie und die Größe des Drehimpulses konstant, aber die tatsächliche Richtung des Drehimpulses muss nicht fest bleiben (wie Sie im Fall der Rotation gesehen haben, die ursprünglich um die Achse mittlerer Länge war). . Konstante | L | gibt
L x 2 + L j 2 + L z 2 = c Ö n s t a n t
Im Hauptachsenrahmen ist es einfach, die Komponenten von zu lösen ω in Bezug auf die Komponenten von L, also eliminieren wir ω aus dem Ausdruck 2 K = L ω , geben
1 ich x x L x 2 + 1 ich j j L j 2 + 1 ich z z L z 2 = c Ö n s t a n t # 2

Die erste Gleichung ist die Gleichung einer Kugel im dreidimensionalen Raum, die vom Drehimpulsvektor eingenommen wird, während die zweite die Gleichung eines Ellipsoids ist:Geben Sie hier die Bildbeschreibung ein

Die obere Abbildung entspricht dem Rotationsfall um die kürzeste Elementachse, die das größte Trägheitsmoment aufweist. Der Schnittpunkt der beiden Flächen besteht nur aus den beiden Punkten an der Vorder- und Rückseite der Kugel. Der Drehimpuls ist auf einen dieser Punkte beschränkt und kann seine Richtung, dh seine Orientierung in Bezug auf das Hauptachsensystem nicht ändern, was eine andere Art zu sagen bedeutet, dass der Schuh seine Orientierung in Bezug auf das System nicht ändern kann Drehimpulsvektor. In der unteren Abbildung dreht sich der Schuh um die längste Achse. Nun wird der Drehimpulsvektor an einem der beiden Punkte rechts oder links eingefangen. Bei der Drehung um die Achse mit dem Trägheitselement dazwischen ist der Schnittpunkt der Kugel und des Ellipsoids jedoch nicht nur ein Paar isolierter Punkte, aber die mit der gestrichelten Linie gezeigte Kurve. Die relative Ausrichtung des Schuhs und des Drehimpulsvektors können und werden sich ändern.

Eine Anwendung des Trägheitstensors sind Videospiele, die Autorennen oder fliegende Flugzeuge simulieren....

Ein weiteres exotisches Beispiel hat mit der Kernphysik zu tun. Obwohl Sie sich Atomkerne wahrscheinlich nur als strukturlose Punkte oder vielleicht winzige Kugeln vorgestellt haben, sind sie oft Ellipsoide mit einer langen Achse und zwei kürzeren, gleichen Achsen. Obwohl ein sich drehender Kern normalerweise seinen Drehimpuls durch Gammastrahlenemission innerhalb eines Zeitraums in der Größenordnung von Pikosekunden abgibt, kann es vorkommen, dass ein deformierter Kern in einen Zustand gerät, in dem ein großer Drehimpuls entlang seiner Längsachse vorhanden ist ist ein sehr stabiler Rotationsmodus. Solche Zustände können Sekunden oder sogar Jahre andauern! (Zur Geschichte gehört noch mehr – das ist das Thema, zu dem ich meine Doktorarbeit geschrieben habe –, aber die grundlegende Einsicht trifft zu, auch wenn die vollständige Behandlung ausgefallene Quantenmechanik erfordert.)

Geben Sie hier die Bildbeschreibung ein

Unsere Analyse ging bisher davon aus, dass die kinetische Energie der Rotationsenergie nicht in andere Energieformen wie Wärme, Schall oder Vibration umgewandelt werden kann. Wenn diese Annahme versagt, wird die Drehung um die Achse mit dem geringsten Trägheitsmoment instabil und wandelt sich schließlich in eine Drehung um die Achse um, deren Trägheitsmoment am größten ist. Dies geschah mit dem ersten künstlichen Satelliten der USA, Explorer i, der 1958 gestartet wurde. Beachten Sie die langen schlaffen Antennen, die dazu neigten, die kinetische Energie in Vibrationen umzuwandeln. Es war so konstruiert, dass es sich um seine Achse mit dem minimalen Trägheitsmoment drehte, aber fast sofort, sobald es im Weltraum war, begann es sich von einem Ende zum anderen zu drehen. Dennoch konnte sie ihren wissenschaftlichen Auftrag erfüllen, der nicht von einer stabilen Orientierung abhing,

Eine verwandte Frage hier auf Physics.SE
Eine verwandte Frage auf MathOverflow

Es gibt eine Antwort auf den MathOverflow-Link, die versucht, das Phänomen zu erklären, aber meiner Meinung nach ist sie für die Situation hier überhaupt nicht zufriedenstellend. Sein Punkt ist, dass es eine Zentrifugalkraft gibt, die dazu neigt, die Masse orthogonal zur Rotationsachse "abzuflachen". Aber das stimmt nicht mit dem Fall (1) überein, der instabil wäre, wenn es nur um die Zentrifugalkraft ginge.
"Beim Drehen um eine beliebige Achse ist die Instabilität der Drehung proportional zur Differenz zwischen den Trägheitsmomenten um die anderen beiden Achsen, wie in den Gleichungen 1-3 oben beschrieben." Hast du dafür eine Referenz? Die Mittelachse ist unglaublich instabil, und die längste und kürzeste Achse sind extrem stabil, und diese Größe unterscheidet sich nur um den Faktor 2.
Ich frage mich dasselbe wie @PeterShor ...
@PeterShor Es ist faszinierend, dass Sie einen Faktor von 2 ansprechen, da Richard Feynman einige interessante Experimente zum Wackeln von sich drehenden Scheiben durchgeführt hat , die genau diesen Faktor beinhalten! Außerdem gab es immer einen Link zum YouTube-Video am Ende meiner Antwort, das die von Ihnen angeforderte Quelle war.
Das Kopfgeld hier zu vergeben - auch wenn es keine besonders befriedigende Antwort ist, versucht es zumindest, die Dinge qualitativ zu erklären, und bietet einige Ideen für mögliche Erklärungen. Akzeptieren Sie jedoch die andere Antwort, da dies eine robuste mathematische Demonstration der Instabilität ist.
@fffred: Ich denke, Terry Taos Antwort auf MathOverflow (die, auf die Sie sich vermutlich beziehen) funktioniert. Für Fall (1) eine Verschiebung der leichten Massen m aus der Rotationsebene würde die Zentrifugalkraft entgegenwirken. Nur im Fall (3) wird diese Verschiebung verstärkt.

Ich habe dieses Problem vor 20 Jahren in einem klassischen Mechanikkurs untersucht, und ich erinnere mich, dass Sie beim Lösen der Bewegungsgleichungen eine imaginäre Exponentialkomponente erhalten, die die Bewegung für jede Achse beschreibt.

Im Fall der Zwischenachse erhalten Sie jedoch eine Multiplikation zweier imaginärer Zahlen im Exponential, was dann im Exponential eine reelle Zahl ergibt, und statt einer sinusförmigen Bewegung beginnt es, rein exponentiell zu werden.

Natürlich führt dies nicht zu einer tatsächlichen exponentiellen Bewegung, da es nichts gibt, um sie aufrechtzuerhalten, so wie die Trägheit die Rotation für eine Weile in den anderen beiden Richtungen aufrechterhält, aber sie wird dadurch schnell instabil.

Intuitiv, warum dies über die Mathematik hinaus instabil werden sollte, habe ich nie fest im Griff. Und ich nehme an, in ähnlicher Weise ist es schwierig, irgendetwas im QM jemals intuitiv zu erfassen, ich habe es nie wirklich versucht ...

Ich denke, für Laien kann dies nur mit Trägheitsmoment und Massenschwerpunkt beschrieben werden. In Beispiel (a) würde das Buch mit horizontaler Rotationsachse gehalten, was ein geringes Trägheitsmoment um andere Achsen ergibt, weil es in dieser Richtung sehr schmal ist. Dies würde die Drehung um eine andere Achse minimieren und das Buch würde sich nur um Achse 1 drehen.

In Beispiel (b) ist das Gegenteil der Fall. Das Trägheitsmoment ist jedoch höher, da das Buch in dieser Richtung am breitesten ist, ist es einfacher, Ihre Finger näher an der durch den Massenmittelpunkt gerichteten Achse zu platzieren, wodurch weniger Drehmoment in andere Richtungen erzeugt wird.

Beispiel (c) würde ein Problem darstellen, da der geringste Versatz von der Mitte ein Drehmoment in andere Richtungen erzeugen könnte. Ich würde annehmen, dass Ihre Hand im Vergleich zur Breite des Buches groß ist und fast immer von der Mittelachse versetzt ist. Aber auch mit kleineren Händen wäre es noch schwierig, da jedes Transekt aufgrund der Länge viel Masse hätte und ein Abwinkeln des Buches ein Drehmomentungleichgewicht erzeugen würde.

Das ist meine Interpretation dieses Problems ohne Verwendung von Mathematik auf Hochschulniveau und offen für Meinungsverschiedenheiten.